¿Un valor infinito para un campo eléctrico?

Considere un sistema de cargas puntuales. Para calcular el valor de un campo eléctrico en un punto, consideramos la contribución del campo eléctrico de todas las cargas en ese punto. Considere la siguiente situación:

Digamos que tenemos un sistema de dos cargas puntuales y quiero calcular el valor del campo eléctrico en el punto donde se encuentra una de las cargas puntuales. Consideraré la contribución de las dos cargas usando la ley de Coulomb:

mi = k q r 2

Cuando hago esto lo que obtengo es mi = desde r = 0 , desde límite X 0 F ( X ) = 1 X 2 = .

Ahora, si tuviéramos que calcular la fuerza que actúa sobre esa carga en ese punto, que viene dada por:

F = q mi , desde mi = , por lo tanto F = y a = dónde a es la aceleración.

¿Puede alguien por favor corregirme?

Dos problemas aquí (1) No hagas que una partícula se vea afectada por su propio campo (2) cuando dices " r = 0 "tienes que elegir un origen, lo cual es importante si tienes dos cargos porque la cantidad relevante será su separación.
@zeldredge (1) ¿puede explicarme por qué una partícula no puede verse afectada por su propio campo? (2) cuando digo r = 0 Me refiero a que la distancia entre la carga y el punto en el que quiero calcular su intensidad de campo es cero. En otras palabras, quiero calcular el campo en el punto en el que se encuentra la carga.
@OmarNagib: la autofuerza eléctrica no es un problema trivial: math.utk.edu/~fernando/barrett/bwald1.pdf
@JerrySchirmer ¿Significa esto que la respuesta presentada por Steven en la que argumentó que no es posible la fuerza propia debido a la tercera ley de Newton es incorrecta?
@OmarNagib: la fuerza propia solo está presente si la carga se está acelerando.

Respuestas (2)

La exposición que das está bien en física clásica. Sin embargo, tenga en cuenta que en la física clásica una partícula no puede ser una partícula puntual, porque algo tiene que llevar la carga en las formulaciones de la física clásica. Entonces, el hecho de que uno encuentre el infinito en r=0 simplemente golpea esta restricción. Uno podría usar el argumento como prueba por reducción al absurdo de que las partículas deberían tener un tamaño.

Las partículas elementales son partículas puntuales pero también son objetos mecánicos cuánticos. El reino de la mecánica cuántica es el reino del principio de incertidumbre de Heisenberg . La ubicación de la partícula elemental es incierta dentro de los límites dados por el HUP. El micromundo de las partículas puntuales tiene reglas diferentes. El valor del campo de un electrón que se encuentra con un positrón se vuelve irrelevante cuando se aniquilan. Todo esto se vuelve matemáticamente riguroso por la solución de las ecuaciones mecánicas cuánticas.

En general, cada vez que la física clásica da infinitos, uno encuentra que la formulación de la mecánica cuántica los elimina. Y la mecánica cuántica es el nivel subyacente de la naturaleza del que emergen todos los campos clásicos y sus ecuaciones .

¿Puede alguien por favor corregirme?

No hay nada que corregir. No estas equivocado.

Digamos que su carga es negativa.

Si pones una carga positiva exactamente sobre la carga negativa con la distancia r = 0 entre ellos, entonces sí, la fuerza es bastante grande. Es más difícil separarlos que si hubiera una distancia mayor. Entonces, podría ocurrir algún tipo de fusión ahora (que es otro tema), ya que la fuerza es tan grande.

Seguramente, no puede tener una distancia exactamente cero, pero su pensamiento teórico es correcto. Considere un dibujo de campo. Donde las líneas de campo están más cerca, el campo es más fuerte. Cuando se encuentran (que en teoría solo será exactamente en la ubicación de la carga), el campo eléctrico sería, en teoría, infinito.

ingrese la descripción de la imagen aquí

De la pregunta elaborada en los comentarios:

¿Por qué esta carga no se ve afectada por su propio campo que en este caso debería ser infinito ya que r = 0 ?

Considere una carga como un planeta. El planeta fluye en el espacio y simplemente está allí. No tira por sí mismo; su gravedad no se acelera a sí misma.

Ahora, si un gran asteroide está cerca, entonces el planeta comienza a atraer este astroide. Se tira en él. Esto también moverá el planeta. De la tercera ley de Newton, la fuerza con la que tira también actúa sobre sí mismo, tirando hacia adelante con exactamente la misma fuerza. Pero en la dirección opuesta.

Ahora, piensa qué pasaría si se jalara a sí mismo con su fuerza gravitatoria. Se impulsará en "alguna" dirección. Pero al mismo tiempo, según la tercera ley de Newton, exactamente la misma magnitud de fuerza lo empujará hacia adelante en la dirección opuesta. No habrá fuerza neta.

Del mismo modo, cuando me levanto de mi propio cabello, no voy a volar. Exactamente la misma fuerza con la que estoy tirando, también me está empujando hacia abajo .

Del mismo modo para cualquier fuerza de cualquier "objeto". Del mismo modo para los cargos. Nada puede ejercer una fuerza neta sobre sí mismo.

Creo que no me expliqué lo suficiente. Olvidémonos de los dos sistemas de carga anteriores y consideremos un sistema físico que consta de una sola carga puntual. ¿Por qué esta carga no se ve afectada por su propio campo que en este caso debería ser infinito ya que r = 0 ?(la distancia entre el punto considerado y la carga es cero). Entonces, ¿puede editar su respuesta para abordar eso?
@OmarNagib Si coloca un cargo sobre la mesa, atraerá otros cargos . Si no hay otros cargos, no pasará nada. Esta es simplemente la tercera ley de Newton. Si te levantas por los cabellos, no levitarás; no importa lo fuerte que seas.
Estaría muy agradecido si elaborara más en detalle cómo se relaciona con la tercera ley de Newton.
@OmarNagib Claro. He hecho una adición a la respuesta. Por favor, hágamelo saber, si esto no lo aclara.
Aunque su argumento es convincente para mí, ¿está seguro de que la respuesta es tan trivial? Digo esto porque me topé con esto: enlace Y @JerrySchirmer me dio este enlace: enlace Entonces, ¿cuáles son sus pensamientos al respecto?
@OmarNagib Muy interesante. Nunca he oído hablar de eso antes. Me parece que esto tiene que ver con que la carga no sea una "esfera rígida" como se suele modelar. Pero solo estoy adivinando aquí; por qué esta fuerza propia supuestamente debería estar allí (cuál es la razón por la que se cree que existe) es una buena pregunta para hacer.
@OmarNagib Acerca de la fuerza propia: physics.stackexchange.com/questions/184695/…